Tamon worked 40 hours this week. Assuming he makes $9 an hour, what is his gross pay?
How much was withheld from his check if his total withholding taxes are 20%? What is
his net pay?

Answers

Answer 1

Answer:

there will be 1 min added


Related Questions

Need help on algebra please

Answers

Answer:

Step-by-step explanation:

Write an equation for the polynomial graphed below

Answers

The equation of the polynomial function graphed is given as follows:

y = -0.125(x³ + x² - 14x - 24).

How to obtain the equation of the polynomial?

From the graph, the roots of the polynomial are given as follows:

x = -3.x = -2.x = 4.

Considering the roots, the linear factors of the polynomial are given as follows:

x + 3.x + 2.x - 4.

Applying the Factor Theorem, the function, as a product of it's linear factors, is given as follows:

y = a(x + 3)(x + 2)(x - 4).

y = a(x² + 5x + 6)(x - 4)

y = a(x³ + x² - 14x - 24).

When x = 0, y = 3, hence the leading coefficient a is obtained as follows:

-24a = 3

a = -0.125.

Hence the function is:

y = -0.125(x³ + x² - 14x - 24).

More can be learned about functions at https://brainly.com/question/24808124

#SPJ1

A positive integer is twice another. The difference of the reciprocals of the two positive integers is frac(1,10). Find the two integers.

Answers

The two integers are 5 and 10.

What do you mean by Integers?

An integer is a number with no decimal or fractional part and it includes negative and positive numbers, including zero.

Integer is a Latin word which means 'whole' or 'intact'. This means integers do not include fractions or decimals.

A number is positive if it is greater than zero, so it said to be positive integers

A number is negative if it is less than zero, so it said to be negative integers.

A number line is a visual representation of numbers on a straight line. This line is used for the comparison of numbers that are placed at equal intervals on an infinite line that extends on both sides, horizontally.

Given:

Let x be the positive number

2x be the other positive number

Find the first integer

[tex]\frac{1}{x} - \frac{1}{2x} = \frac{1}{10}[/tex]

[tex]\frac{2x - x}{2x^2} = \frac{1}{10}[/tex]

[tex]\frac{x}{2x^2} = \frac{1}{10}[/tex]

[tex]\frac{1}{2x} = \frac{1}{10}[/tex]

2x = 10

x = 5

Therefore, the first positive integer is 5.

Find the other integer

2x = 2(5) = 10

Therefore, the other integer is 10.

To check:

[tex]\frac{1}{x} - \frac{1}{2x} = \frac{1}{10}[/tex]

[tex]\frac{1}{5} - \frac{1}{10} = \frac{1}{10}[/tex]

[tex]\frac{2}{10} - \frac{1}{10} = \frac{1}{10}[/tex]

[tex]\frac{1}{10} = \frac{1}{10}[/tex]

To know more about Integers visit:

brainly.com/question/1768254

#SPJ1

Rob loaded 9 trucks in 1 hour find his loading speed per hour

Answers

Answer:

9 trucks/hour

Step-by-step explanation:

If he loaded 9 trucks in one hour, his loading speed is 9 trucks per hour.

Xin is going to invest in an account paying an interest rate of 5.8% compounded annually. How much would Xin need to invest, to the nearest hundred dollars, for the value of the account to reach $2,290 in 16 years?

Answers

The amount that should be invested in order to have $2,290 in 16 years is $929.11.

What is compound interest?

The interest on a deposit calculated using both the initial principle and the accrued interest from prior periods is known as compound interest. In other words, compound interest is interest that is earned on interest.

Here, we have

Given: Xin will invest in an account paying an interest rate of 5.8% compounded annually.

Amount to invest = Future value / (1 + r)ᵗ

Where:

r = interest rate

t = time

X = $2,290 / (1 + 0.058)¹⁶

X = 2,290 / 2.46474

X = $929.11

Hence, the amount that should be invested in order to have $2,290 in 16 years is $929.11.

To learn more about compound interest the given link

https://brainly.com/question/24274034

#SPJ1

a restaurant uses square tables with sides of length 1.3m, and round tablecloths with diameter 2m. Determine the percentage of each tablecloth which overhangs its table.

Answers

Answer:

46.21%

Step-by-step explanation:

The area of the table top is 1.3 x 1.3 = 1.69 m²

The area of the circular tablecloth is given by πr² where r is the radius of the table cloth

Give diameter of tablecloth is 2m, radius r = 2/2 = 1 m

Area of tablecloth = π · 1² = π

So the excess area of the tablecloth which overhangs

= π - 1.69 = 1.45 m²  (taking π = 3.14)

So the fraction of the tablecloth that overhangs
= Excess area of tablecloth ÷ total area of tablecloth
= 1.45 /π

= 0.4621

As a percentage this would be
0.4621 x 100

= 46.21%

PLEASE HELP!!! PLEASE IM BEGGING SOMEONE TO HELP ME GHIS IS DUE TOMORROW

Answers

Answer:

D

Step-by-step explanation:

It's the most random and produces no bias.

Answer:

D

Step-by-step explanation:

A certain antihistamine is often prescribed for allergies. A typical dose for a 100​-pound person is 22 mg every six hours. Complete parts​ (a) and​ (b) below.
b. This antihistamine also comes in a liquid form with a concentration of 12.3 ​mg/ 6 mL. Following the prescribed​ dosage, how much liquid antihistamine should a 100​-pound person take in a​ week?

Answers

300.5 ml of antihistamine is needed for a 100​-pound person to take in a​ week

What is an equation?

An equation is an expression that shows how numbers and variables are linked together using mathematical operations such as addition, subtraction, multiplication and division.

1 day = 24 hours

1 week = 7 days = 24 hours per day * 7 days = 168 hours

A typical dose for a 100​-pound person is 22 mg every six hours. Hence for one week:

Dosage in one week = (22 mg per 6 hours) * 168 hours = 616 mg

Antihistamine also comes in a liquid form with a concentration of 12.3 ​mg/ 6 mL. For 616 mg:

Amount of antihistamine = 616 mg / (12.3 ​mg/ 6 mL) = 300.5 ml

300.5 ml of antihistamine is needed

Find out more on equation at: https://brainly.com/question/2972832

#SPJ1

Which equation represents the graph?

a graph of a line that passes through the points 0 comma negative 1 and 1 comma negative 4

y equals negative one third times x minus 1
y = −3x − 1
y equals negative one third times x plus one third
y equals negative 3 times x plus one third

Answers

The equation that represents the graph of a line passing through the points (0, -1) and (1, -4) is y = -3x - 1.

What is graph?

A graph is a visual representation of data or information that shows the relationship between different variables or quantities. Graphs can take many forms, including bar graphs, line graphs, scatterplots, and pie charts, among others. They are widely used in many fields, including mathematics, science, economics, and social sciences, to help visualize and analyze data, identify patterns, and communicate findings to others in a clear and concise manner. By representing complex information in a simple, easy-to-understand format, graphs can provide valuable insights and help us make informed decisions.

The equation that represents the graph of a line passing through the points (0,-1) and (1,-4) is:

y = -3x - 1

To find the equation of a line given two points, you can use the slope-intercept form:

y - y1 = m(x - x1)

where m is the slope of the line and (x1, y1) is one of the given points.

First, we can find the slope of the line:

m = (y2 - y1) / (x2 - x1)

= (-4 - (-1)) / (1 - 0)

= -3

Next, we can choose one of the given points and substitute its coordinates and the slope into the slope-intercept form:

y - (-1) = -3(x - 0)

y + 1 = -3x

y = -3x - 1

Therefore, the equation that represents the graph is y = -3x - 1.

Option A, y = -1/3x - 1, has a slope of -1/3, which is not the slope of the line passing through the given points.

Option C, y = -1/3x + 1/3, also has an incorrect slope of -1/3, and it also does not pass through the point (0,-1).

Option D, y = -3x + 1/3, has the correct slope of -3, but it does not pass through the point (0,-1).

Lear more about equation of line click here:

https://brainly.com/question/25969846

#SPJ1

The value of a motorcycle changes according to the equation V=5,000(1.03)t^, where V= value in dollars and t= time in years.

Use the dropdowns to complete the statements.


In the equation, the number 5,000 represents the ________ of the motorcycle. The value of the motorcycle is __________ at a rate of ________ per year.

Answers

In the equation, the number 5,000 represents the initial value of the motorcycle. The value of the motorcycle is increasing at a rate of 3% per year.

What are Exponential Functions?

Exponential functions are functions where the independent variable, x is in the exponent.

The given exponential equation is,

V = 5,000 (1.03)^t

Here V represents the value of the motorcycle in t years.

When t = 0,

V = 5000 (1.03)⁰ = 5000

So 5000 represents the initial value of the motorcycle.

V = 5,000 (1 + 0.03)^t

At t = 0, V = 5000

At t = 1, V = 5000 (1.03)¹ = 5150

At t = 2, V = 5000 (1.03)² = 5304.5

So the value of the motorcycle is increasing.

The rate is 0.03 or 3%.

Hence, 5,000 represents the initial value of the motorcycle and the value of the motorcycle is increasing at a rate of 3% per year.

Learn more about Exponential Functions here :

https://brainly.com/question/14355665

#SPJ1

Can someone please help me do this

Answers

The vertices of the image of triangle ABC are A''(x, y) = (- 6, 8), B''(x, y) = (- 8, 4) and C''(x, y) = (- 4, 4).

How to determine the image of a triangle set on Cartesian plane

Any triangle can be generated by three points that are not collinear, in this problem we need to determine the image of a triangle, which is the result of two rigid transformations: (i) Reflection over the y-axis, (ii) Dilation centered at the origin with a scale factor of 2, whose definitions are shown below:

Reflection over the y-axis:

(x, y) → (- x, y)

Dilation centered at the origin with a scale factor of 2:

(x, y) → (2 · x, 2 · y)

If we know that A(x, y) = (3, 4), B(x, y) = (4, 2), C(x, y) = (2, 2), then the image of the vertices are determined below:

Reflection over the y-axis

A'(x, y) = (- 3, 4), B'(x, y) = (- 4, 2), C'(x, y) = (- 2, 2)

Dilation centered at the origin with a scale factor of 2

A''(x, y) = (- 6, 8), B''(x, y) = (- 8, 4), C''(x, y) = (- 4, 4)

To learn more on rigid transformations: https://brainly.com/question/17952620

#SPJ1

estimate the population in the year 2040

Answers

well, in 2007 it was 12000, so initially that's what it was, and in 2019 it went to 23000, so that's 12 years later, and in 2040, that'll be 33 years later.

[tex]\qquad \textit{Amount for Exponential Growth} \\\\ A=P(1 + r)^t\qquad \begin{cases} A=\textit{accumulated amount}\dotfill & \$ 23000\\ P=\textit{initial amount}\dotfill &12000\\ r=rate\to r\%\to \frac{r}{100}\\ t=years\dotfill &12\\ \end{cases} \\\\\\ 23000 = 12000(1 + \frac{r}{100})^{12}\implies \cfrac{23000}{12000} =\left(1+ \cfrac{r}{100} \right)^{12} \\\\\\ \cfrac{23}{12}=\left(\cfrac{100+r}{100} \right)^{12}\implies \sqrt[12]{\cfrac{23}{12}}=\cfrac{100+r}{100}[/tex]

[tex]100\sqrt[12]{\cfrac{23}{12}}=100+r\implies 100\sqrt[12]{\cfrac{23}{12}}-100=r\implies \boxed{5.57\approx r} \\\\[-0.35em] ~\dotfill\\\\ \qquad \textit{Amount for Exponential Growth} \\\\ A=P(1 + r)^t\qquad \begin{cases} A=\textit{accumulated amount}\\ P=\textit{initial amount}\dotfill &12000\\ r=rate\to 5.57\%\to \frac{5.57}{100}\dotfill &0.0557\\ t=years\dotfill &\stackrel{year~2040 }{33}\\ \end{cases} \\\\\\ A \approx 12000(1 + 0.0557)^{33} \implies \boxed{A \approx 71782}[/tex]

When you use the distance formula you are building a right triangle whose ___ connects two given points

Answers

Answer:

Step-by-step explanation:

When we use the distance formula for two points ( x, y ) and ( x', y¹ ) Then formula is (distance)² = (x - x¹)² + (y - y¹)² This is quite similar to the formula used in a right angle triangle

x^{2}-2x+ and completing the square ( Algebra 2 )

Answers

x^2 - 2x + 1 is the required quadratic equation using the completing the square

Perfect square trinomials using the completing the square

Given the quadratic expression below

x^2 - 2x

We need to determine the constant that will make the expression a perfect square.

The constant will be the half of the square of coefficient of 'x'

Coefficient of 'x' = -2

Half of the coefficient of 'x' = -2/2 = -1

Square of the coefficient = (-2/2)^2

Square of the coefficient = 1

Hence the complete quadratic expression using the completing the square is x^2 - 2x + 1

Learn more on completing the square here: https://brainly.com/question/13981588

#SPJ1

Counting back from 18, what number follows 17?​

Answers

Answer:

16

Step-by-step explanation:

As the question states, we are counting downwards which means with every interval we are going back -1. Therefore, 17 - 1 = 16

Sets A and B are subsets of the universal set U.
These sets are defined as follows.
U= {f, k, m, q, x,y}
A={f, k, m, y}
B = {f, m,q}
Find the following sets.
Write your answer in roster form or as Ø.
(a) AUB' =
(b) A' B' =

Answers

(a) The union set is AUB'={f, k, m, x, y}.

(b) The union set is A'UB'={q,k, x, y}.

What is the union of two sets?

The union of two sets is also a set. This set contains all the elements of both two sets.

The universal set is U= {f, k, m, q, x,y} and two subsets are A={f, k, m, y} and B = {f, m,q}.

(a)

B' is the complementary set of B. So, find the elements of B'=U-B.

B'= {f, k, m, q, x,y}-{f, m,q}

={k, x,y}

Now, find the union set AUB'.

AUB' ={f, k, m, y}U{k, x,y}

={f, k, m, x, y}

Therefore, the required answer is AUB'={f, k, m, x, y}.

(b)

Also, A' is the complementary set of A. So, find the elements of A'=A-B.

A'= {f, k, m, q, x,y}-{f, k,m, y}

={q,x}

Now, find the union set A'UB'.

A'UB' ={q,x}U{k, x,y}

={q,k, x, y}

Therefore, the required answer is A'UB'={q,k, x, y}.

To learn more about the union of sets visit this link

https://brainly.com/question/29078120

#SPJ1  

One cubic foot holds 7.48 gallons of​ water, and 1 gallon of water weighs 8.33 pounds. How much does 4.4 cubic feet of water weigh in​ pounds? In​ tons?

Answers

Answer:

4.4 cubic feet of water weighs approximately 273.5016 pounds or 0.13676 tons.

Step-by-step explanation:

To find the weight of 4.4 cubic feet of water in pounds, we need to find the number of gallons it contains and then multiply it by the weight of one gallon of water.

1 cubic foot = 7.48 gallons

So, 4.4 cubic feet = 4.4 * 7.48 = 32.912 gallons

And the weight of 32.912 gallons of water in pounds is:

32.912 gallons * 8.33 pounds/gallon = 273.5016 pounds

To convert this weight to tons, we divide the weight in pounds by 2000:

273.5016 pounds / 2000 = 0.13676 tons

So, 4.4 cubic feet of water weighs approximately 273.5016 pounds or 0.13676 tons.


2. What is the equation of the parabola that passes
through the points (-1,0), (7, 0) and (3,-16)?
3. What is the equation of the parabola that passes
through the points (-4,9), (-3, 2) and (0,-7) and (4,9)?

Answers

The equations of the parabolas are y = x² - 6x - 7 and y = x² - 7

How to determine the equation of the parabolas

Parabola 1

Given that

The points (-1,0), (7, 0) and (3,-16)

The equation is represented as

y = ax² + bx + c

So, we have

a - b + c = 0

49a + 7b + c = 0

9a + 3b + c = -16

Make c the subject in a - b + c = 0

c = b - a

So, we have

49a + 7b + b - a = 0

9a + 3b + b - a = -16

48a + 8b = 0

8a + 4b = -16

Multiply 8a + 4b = -16 by 2

48a + 8b = 0

16a + 8b = -32

Subtract

32a = 32

So, we have

a = 1

Recall that 48a + 8b = 0

So, we have

48 + 8b = 0

This gives

8b = -48

Divide

b = -6

Also, we have

c = b - a

This gives

c = -6 - 1

c = -7

So, the equation is

y = x² - 6x - 7

Parabola 2

Given that

The points (-4,9), (-3, 2) and (0,-7) and (4,9)

The equation is represented as

y = ax² + bx + c

So, we have

16a - 4b + c = 9

9a - 3b + c = 2

c = -7

16a + 4b + c = 9

The equations become

16a - 4b = 16

9a - 3b = 9

16a + 4b = 16

Add (1) and (3)

32a = 32

So, we have

a = 1

Subtract (1) and (3)

-8b = 0

So, we have

b = 0

So, the equation is

y = x² - 7

Read more about parabola at

https://brainly.com/question/1480401

#SPJ1

What number is 7% smaller than 74

Answers

Answer: 68.82

Step-by-step explanation:

74 * (1- 7%) and solve

= 68.82

Write an equation to describe the sequence below. Use n to represent the position of a term
In the sequence, where n = 1 for the first term.
-25, -50, -100,...
Write your answer using decimals and integers.
a,
Blank 1:
Blank 2:
Submit
C
C

Answers

An equation to show the sequence is -25n.

What is a geometric series?

When all the terms of the geometric sequence are added, than that expression is known as geometric series.

The sum of terms of a geometric sequence;

Now, lets suppose its initial term is   , multiplication factor will be r

and let it has total n terms, then, its sum is given as:

[tex]S_n = \dfrac{a(r^n-1)}{r-1}[/tex]

(sum till nth term)

We are given that;

The first term of sequence=-25

D= -50-(-25)

=-25

Now,

an = a + (n-1)d

=-25+(n-1)-25

=-25-25n+25

=-25n

Therefore, the answer of the sequence will be -25n.

Learn more about sum of terms of geometric sequence visit:

brainly.com/question/1607203

#SPJ9

Which statements correctly describe how the graph of the geometric sequence below should appear?

640, 160, 40, 10, ...

Select two options.


Answers

The true statements are

The domain will be the set of natural numbers.The graph will show exponential decay.

How to determine the true statement

From the question, we have the following parameters that can be used in our computation:

The sequence:

640, 160, 40, 10, ...

Because the current term is less than the previous term, then it represents a decay function

And also, the domain will be the set of natural numbers

This is so because the input values are 1, 2, 3 and so on

Read more about exponential function at

https://brainly.com/question/2396210

#SPJ1

Complete question

Which statements correctly describe how the graph of the geometric sequence below should appear?

640, 160, 40, 10, ...

Select two options.

The graph will show exponential growth.

The graph will appear linear.

The domain will be the set of natural numbers.

The range will be the set of natural numbers.

The graph will show exponential decay.

Answer:

C & D

Step-by-step explanation:

Timothy, Laurence and Mike went to a bookstore to buy some books. The ratio of the number of books bought by Laurence and Mike were 2: 7. Timothy and Laurence bought a total of 20 books while Timothy and Mike bought a total of 35 books. How many books did Timothy buy?

Answers

Answer:

[tex]\mbox{\large \textsf{Timothy bought 14 books}}[/tex]

Step-by-step explanation:

Let's use first letters of names to represent the number of books bought by that person. This makes it easier to explain.

Therefore, the books bought by each of the three persons are T(imothy), L(aurence) and M(ike)

We will now convert each of the word description into math equations and solve

We are given that the ratio of the number of books bought by Laurence and Mike were 2: 7

We can write this as:
[tex]\dfrac{L}{M} = \dfrac{2}{7}\\\\[/tex]

Multiplying both sides by M:
[tex]L = \dfrac{2}{7}M\cdots\cdots(1)[/tex]  

Timothy and Laurence bought a total of 20 books can be represented as
[tex]T + L = 20\cdots\cdots(2)[/tex]

Timothy and Mike bought a total of 35 books can be represented as
[tex]T + M = 35\cdots\cdots(3)[/tex]


Eq (3) - Eq (2):

[tex]T + M - (T + L) = 35 - 20\\\\T + M - T - L = 15\\\\M - L = 15 \cdots\cdots (4)[/tex]

Substituting for [tex]\displaystyle L = \frac{2}{7}M[/tex]  from equation (1) into equation (4) we get

[tex]M - \dfrac{2}{7}M = 15\\\\\dfrac{5}{7}M = 15\\\\M = \dfrac{7}{5} \times 15\\\\M = 21\\\\[/tex]

Given [tex]\displaystyle M = 21[/tex] using equation (3), [tex]\displaystyle T + M = 35[/tex] we get

[tex]T + 21 = 35\\\\T = 35 - 21\\\\\textrm{or}\\\\T = 14[/tex](Answer)

If we wanted to find out how many books Laurence bought use Eq(1)
[tex]L = \dfrac{2}{7}M\\\\L = \dfrac{2}{7} \times 21\\\\L = 6\\\\[/tex]

product p of three numbers x ,y,and z

Answers

Answer:

p = x(y)(z)

Step-by-step explanation:

product is multiplication.

multiplication has no specific order.

The product of three numbers x, y, and z can be calculated as follows:

p = x * y * z

Where x, y, and z are the three numbers whose product we want to find.

please help meee

here is the picture

Answers

Answer:

f(-2) = 46

f(-1) = 17

f(0) = 2

f(1) = 1

f(2) = 14

Step-by-step explanation:

F(-2) = 46

F(-1)= 17

F(0)= 2

F(1)= 1

F(2)= 14

Sheldon wants to buy two shirts at the store.

They were each originally $14.99, but one of the shirts is on sale this week for 10% off.

A 6% sales tax is applied to the total cost of the shirts.

Sheldon writes an equation to calculate the total cost, n, of his purchase.


0.90 • 14.99 + 14.99 • 1.06 = n


Which statement describes Sheldon’s equation?

Responses

His equation correctly represents the total cost of his order.

According to his equation, the discount will be applied to both shirts.

According to his equation, sales tax will only be added to one shirt.

He should have multiplied the cost of the shirts by 0.0 to calculate tax.

Answers

His equation correctly represents the total cost of his order is the statement describes Sheldon’s equation

What is Equation?

Two or more expressions with an Equal sign is called as Equation.

Sheldon wants to buy two shirts at the store.

They were each originally $14.99, but one of the shirts is on sale this week for 10% off.

A 6% sales tax is applied to the total cost of the shirts.

0.90 • 14.99 + 14.99 • 1.06 = n

The first term in the equation, 0.90 • 14.99, represents the cost of the shirt that is on sale.

The 10% discount is applied to the original price of $14.99, which is why we multiply by 0.90.

This term calculates the discounted price of the first shirt.

The second term in the equation, 14.99 • 1.06, represents the cost of the second shirt, plus the 6% sales tax that is applied to both shirts.

This term calculates the total cost of the second shirt with sales tax.

Hence, His equation correctly represents the total cost of his order is the statement describes Sheldon’s equation

To learn more on Equation:

https://brainly.com/question/10413253

#SPJ9

The vertex of this parabola is at (5,-4). Which of the following could be its
equation?

Answers

The possible equation of the parabola is y = (x - 5)^2 - 4

How to determine the possible equation

from the question, we have the following parameters that can be used in our computation:

Vertex = (5, -4)

The vertex form of a parabola is given by:

y = a(x - h)^2 + k

where (h, k) is the vertex of the parabola and "a" is a constant that determines the shape of the parabola.

We are given that the vertex of the parabola is (5,-4). Substituting these values into the vertex form, we get:

y = a(x - 5)^2 - 4

Let a = 1

So, we have

y = (x - 5)^2 - 4

Hnce, the possible equation is y = (x - 5)^2 - 4


Read more about parabola at

https://brainly.com/question/1480401

#SPJ1

In the diagram, segment AD bisects angle BAC.
Given the following segment lengths,
find the value of x.
Round to the nearest tenth.
AB= 23
AC = 18
Show all your work.

Answers

Answer: x ≅ 11.2

Step-by-step explanation:

We can set up two equations:

Let y = measure of <BAD = measure of <CAD

then:

sin y = x/23

sin y = (20-x)/18

Since both of these are sin y, we can set them equal to each other:

x/23 = (20-x)/18

.: 18x = 460 - 23x

41x = 460

.: x ≅ 11.2

Answer:

x = 11.2

--------------------

Use angle bisector theorem. It states that:

An angle bisector of an angle of a triangle divides the opposite side into two parts that are proportional to the other two sides of the triangle.

Apply this to the given triangle:

AB/AC = BD/CD23/18 = x / (20 - x)

Cross-multiply and solve for x:

23(20 - x) = 18x460 - 23x = 18x460 = 41xx = 460/41x = 11.2 (rounded)

Endpoints of a diameter are (−5,−11) and (−1,1) Find the standard form of the equation for the circle with the following properties.

Answers

Answer:

Standard form of the equation for the circle

(x + 3)² + (y + 5)² = 40

Step-by-step explanation:

The equation of a circle in standard form is
[tex]\boxed{(x - a)^2 + (y-b)^2= r^2}[/tex]

where (a, b) is the center of the circle and r is the radius

To find center of circle

The end points of the diameter are (-5, -11) and (-1, 1)

The center of the circle is midway between these two points

The x-coordinate of the midpoint = (-5 + -1)/2 = -6/2 = -3

The y-coordinate of the midpoint is (-11 + 1) /2 = -10/2 = -5

So the center of the circle is at (-3, -5)

To find the radius,

Calculate the distance from (-3, -5) to any of of the endpoints.

Let's take the endpoint (-1, 1) and find its distance from (-3, -5)

The distance between any two points (x₁, y₁) and (x₂, y₂) is calculated from the formula


[tex]d^2 = {(x_{2} - x_{1})^2 + (y_{2} - y_{1})^2[/tex]

Substituting
(x₁, y₁) = (-1, 1)
(x₂, y₂)  = (-3, -5)

and r for distance

we get

[tex]r^2 = {(-3 - (-1))^2 + (-5 - 1)^2[/tex]

[tex]r^2 = {(-2)^2 + (-6)^2}[/tex]

[tex]r^2 = {{4} + {36}}[/tex]

[tex]r^2 = {40}[/tex]

So the center (a, b) is (-3, -5) and r² =40

Plugging this into the circle equation:
(x - (-3))² + (y - (-5))² = 40

(x + 3)² + (y + 5)² = 40

Help me with this question……

Answers

Given,

y=55cm
w=73 cm

Using Pythagoras Theorem,

[tex]w^{2}[/tex] = [tex]y^{2} + x^{2}[/tex]

73² = 55² + x²

x= √3025

x = 48cm

Answer= 48cm


The total cost (in dollars) of producing x food processors is C(x) = 2400 + 40x - 0.4^2
find the exact cost of producing the 31st food processor

Answers

From given quadratic equation:

Exact Cost of 31st food processor = $15.6

Approx. cost of 31st food processor  =  $15.2

What is a quadratic equation?

The polynomial equations of degree two in one variable of type f(x) = ax2 + bx + c = 0 and with a, b, c, and R R and a 0 are known as quadratic equations. It is a quadratic equation in its general form, where "a" stands for the leading coefficient and "c" for the absolute term of f. (x).

It is given that the quadratic equation has two roots. Roots might have either a real or imaginary nature.

The given quadratic equation for the cost of producing x food processors is:

 C(x) = 2400 + 40x - 0.4x²

a) The exact cost of producing the 31st food PROCESSOR is:

cost of 31 food processors - cost of 30 food processors = C(31) - C(30)

= (2400 + 40*31 - 0.4*31²) - (2400 + 40*30 - 0.4*30²)

= 3255.6 - 3240 = $15.6

b) The marginal cost at x = 31

Differentiate the quadratic equation

C'(x) = 40 - 0.8x

Marginal cost = C'(31) = 40 - 0.8*31 = $15.2

So the cost of producing the 31st food processor is approx $15.2.

Therefore from the given quadratic equation:

Exact Cost of 31st food processor = $15.6

Approx. cost of 31st food processor  =  $15.2

To learn more about quadratic equations, follow the link.

https://brainly.com/question/28038123

#SPJ1

Other Questions
Question- what type of circuit is in the diagram? Calculate the area of a triangle XYZ withangle X = 90, XZ = 15 cm and XY = 12 cm As a citizen living in a farming community, how would it benefit you to have a representative who is on the House Committee on Agriculture? What could you do to further influence the work of the committee? explain kepler's laws of ellipse a baseball is thrown by the center fielder (from shoulder level) to home plate it is caught (on the fly at shoulder level) by the catcher. at what point is the magnitude of the acceleration at a minimum? On August 31, 2024, the general ledger of Ballet Shoes shows a balance for cash of $7,824. Cash receipts yet to be deposited into the checking account total $3,281. The companys balance of cash does not reflect a debit card payment for the purchase of postage expense ($87) or an electronic funds transfer for rent expense ($1,305). The bank statement also revealed the company recorded a debit card transaction for supplies of $412, but the actual amount was $512. These amounts are included in the balance of cash of $3,051 reported by the bank as of the end of August.Required:1. Prepare a bank reconciliation to calculate the correct ending balance of cash on August 31, 2024.2. Record the necessary entry(ies) to adjust the balance for cash. according to the laws of intestate succession, when no surviving heirs can be found, then proceeds from the estate go to what were the techniques of the Sands of Maharashtra When this person was just a few weeks old, he and his mother were kidnapped. His mother was sold as a slave PLEASE HELP!!!!!!!!!1. y=-x^2+2 what is theDomain: Range: Vertex: Axis of symmetry: y intercepts:2. y=x^2-6x+3 What is the: Domain: Range: Vertex: Axis of symmetry: y intercepts:3: y=-2x^2-8x-5 What is the: Domain: Range: Vertex: Axis of symmetry: y intercepts:4. f(x)=-x^2+1 Whats the: Domain: Range: Vertex: Axis of symmetry: y intercepts:5. f(x)=-2x^2+8x-3 Whats the:Domain: Range: Vertex: Axis of symmetry: y intercepts:6. f(x)=2x^2+8x+1 whats the:Domain: Range: Vertex: Axis of symmetry: y intercepts: A riff is a repeated chord progression.A. False, a riff is an ostinato figure.B. False, a riff is a melodic phrase.C. TrueD. False, a riff can be an ostinato figure, a repeated chord progression or a melodic phrase. When the assumption of all the possible outcomes of an experiment are equally likely to occur is used as a basis for assigning probabilities, the approach is called asSelect one:a. Conditional Methodb. Relative Frequency Methodc. Subjective Methodd. Classical Method Question 6 of 20How did the outcome of World War I contribute to the development of theCold War?A. It produced dozens of new democratic states in Eastern Europe,which allied with the United States.B. It devastated the Soviet Union's military, allowing communists toseize control of its government.C. It divided the Ottoman Empire's territory evenly between the UnitedStates and the Soviet Union.D. It resulted in the United States andthe Soviet Union developinginto the world's most powerful states. Leanne's Nursery buys crates of firewood to deliver to customers over the winter. Each cratemeasures 4 feet long, 3.5 feet wide, and 4 feet high. How much storage space does the nurseryneed for the wood if 60 crates are delivered in an order?OA 675 cubic feetOB 3,375 cubic feetOC 3,937.5 cubic feetOD 3,360 cubic feet What is the name for the process of determining the concentration of a solution by slowly adding a known solution to it using a buret? senator elizabeth warren read a letter written by _____ during a confirmation hearing. The following response is one of the best examples of handling feedback. as we learned in the course, that receiving feedback is a skill that employees can be trained to handle this type of feedback. the employee recognizes the mistake and is asked to elaborate without blaming others. in addition, employees asked for advice beyond their job role. this allows both the person and the supervisor to feel comfortable sharing any feedback and have the potential to be a great mentor. being a mentor changes that dynamic of being overly concerned about how the person would take any feedback. asking for more personal experience type of feedback from supervisor seems like a great way to build relationship. also, the employee was more specific and wanted to know at what instance the mistake occurred. this is one of the three fundamentals of three effective feedback content: specific behaviors. in contrast, there are only few weaknesses. one of the weaknesses would be not taking any note while responding to the supervisor. in the future, you always want to be able to reference back and keep track of information. this also allows you to keep track of your progress and evaluate it as well. another, should do is be more proactive and ask if we can set-up a regular follow-up meeting to discuss moving forward. Jared spends a lot of time on the phone. Which is MOST likely to cause him neck pain? organizational behavior refers to: group of answer choices the assessment of an organization's performance. the study of organizational ethics and values. the study of individual mannerisms and group dynamics in organizations. the steps taken by an organization to address failures. Hydrogen iodide decomposes according to the equation shown below. The second order rate constant for this reaction is 1.6 x 10-3 M-1*s-1 at 700 oC. If the initial concentration of HI is 5.1 x 10-2 M, how many seconds will it take for the concentration to be reduced to 4.9 x 10-3 M at 700 oC?